Which is the equation of the line with slope o passing through the point (-3,-1)?

Answers

Answer 1

9514 1404 393

Answer:

  y = -1

Step-by-step explanation:

We assume you want the line with a slope of zero. That is a horizontal line, so y is a constant. In order to make the line go through the point with y=-1, the equation of the line is ...

  y = -1


Related Questions

A carpenter can make at most 20 tables and at most 30 chairs per day. Each table requires 3 hours of labor and each chair 2 hours of labor. The maximum total hours of labor that the carpenter has at his disposal is 96.

Answers

Answer:

Step-by-step explanation:

Can some solve this question for me plz

Answers

I think 'D' is the correct answer.

Are the following triangles similar by the angle-
angle similarity theorem?
A.No, the triangles don't have three angles.
B.Yes, one of the triangles is smaller than
the other.
C.Yes, two pairs of congruent angles are
marked.
D.No, the triangles don't share two pairs of
congruent angles.

Answers

Answer:

D. No, the triangles don't share two pairs of congruent angles.

Step-by-step explanation:

Two triangles can only be proven similar from the AA (Angle-Angle) similarity theorem if it can be deduced that they share two congruent angles.

In this case, both triangles share one angle, however no other angles can be deduced to be congruent. Therefore, the triangles cannot be proven similar from the AA similarity theorem because they do not share two pairs of congruent angles (corresponds with answer choice D).

Answer:

D.No, the triangles don't share two pairs of congruent angles.

Step-by-step explanation:

We know that angle M equals angle H  and that angle I equals the unmarked angle.

We do not know that angle N equals angle O so we cannot state that the triangles are similar

Could someone help me solve this please.

Answers

Answer: maybe 20 % at least because it might me adding

Omar keeps his sneaker collection carefully arranged on the floor of his closet. 8 pairs of
sneakers fit perfectly side-by-side from one end of the closet to the other. The closet is 60
inches wide.
How wide is each pair of sneakers?

Answers

Answer:

7.5 inches wide. I wasnt wrong. for a second i thought it was asking for the width of each individual sneaker.

Answer:

each pair of sneakers are 7.5 inches wide

Where did this part came from how did this value came from

Answers

Answer:

Sin (theta) has the same value for the both value of theta, so both applies.

Answered by GAUTHMATH

PLEASE ANSWER!!
What is the remainder for the synthetic division problem below?
2/ 3 1 2 -7
A. 25
B. 17
C. -29
D. -39

Answers

A: 25.

Explanation: Check the attached image.

For synthetic division, you just need to multiply the 1st number of the polynomial by the divisior, and then, add it up to the next number; then, the coefficient will be multiplied by the divisor, and so on and so forth until you reach the last number... that last coefficient at the end is the reminder that you've been asked for

Evaluate the following expressions using the chip method. SHOW ALL WORK!!!

Answers

Answer:

a. -7 b. -20c. 7

Step-by-step explanation:

a. -9+2, in this case, it is -7 because you take the bigger number and subtract it by the lower number. If the bigger number is negative your answer will be negative, if the bigger number is positive it will be positive it is just really a basic subtraction problem just add the sign.b. In multiplication +++=+ ++-=- and a -+-=+ do your problem without thinking about the signs and then add the signs with the formula I showed you.c. ---=+

Hope this helps :)!

If the number of observations for each sample is 150 units, what is the 3-sigma upper control limit of the process

Answers

Complete Question

Complete Question is attached below

Answer:

[tex]UCL= 0.25[/tex]

Step-by-step explanation:

From the question we are told that:

Sample size[tex]n=150[/tex]

Sample Variants [tex]s=7[/tex]

Sigma control limits  [tex]Z = 3[/tex]

Therefore

Total number of observations is Given as

[tex]T_o=n*s[/tex]

[tex]T_o=150 *7[/tex]

[tex]T_0=1050[/tex]

Generally

Summation of defectivee

[tex]\sum np=23+34+15+30+25+22+18[/tex]

[tex]\sum np= 167[/tex]

Generally the equation for P-bar is mathematically given by

[tex]P-bar=\frac{\sum np}{T_o}[/tex]

[tex]P-bar=\frac{167}{1050}[/tex]

[tex]P-bar=0.16[/tex]

Therefore

[tex]Sp=\sqrt{\frac{P-bar(1-P-bar)]}{ n}}[/tex]

[tex]Sp=\sqrt{\frac{[0.159(1-0.159)]}{150}}[/tex]

[tex]Sp=0.03[/tex]

Generally the equation for 3-sigma upper control limit of the process is mathematically given by

[tex]UCL = P-bar + Z*Sp[/tex]

[tex]UCL= 0.16 + 3*0.03[/tex]

[tex]UCL= 0.25[/tex]

I’m new and i need help!!
Please help me of you know the answers.

Answers

Answer:

2, x, 2x

Step-by-step explanation:

A) 2

They are even numbers, so it is going to be 2.

B) x

It is going to only be x because the exponents are even and odd.

C) 2x

It is going to be 2x because the numbers are even but the exponents are even and odd.

Hope this helps.

Choose ALL of the following functions that represent exponential decay. f(x) = 5(2/3)^x

Answers

Answer:

exponential decay

Step-by-step explanation:

The function is in the form

y = ab^x  where a is the initial value and b is the growth or decay rate

If b >1 then it is growth

b < 1 then it is decay

f(x) = 5(2/3)^x

a = 5

b = 2/3

2/3 <1 so it is decay


Two buses leave towns 576 kilometers apart at the same time and travel toward each other. One bus travels 12
h
slower than the other. If they meet in 3 hours, what is the rate of each bus?
km
Rate of the slower bus:

Rate of the faster bus:

Answers

Answer:

Rate of slower bus; 90 km/h

Rate of faster bus; 102 km/b

Step-by-step explanation:

We know that formula do distance is;

Distance = speed/time

We are told that One bus travels 12h slower than the other.

Let speed of slower bus be x.

Thus;

Speed of faster bus = x + 12

Speed of slower bus = x

After 3 hours, distance by faster bus = 3(x + 12)

Speed of slower bus = 3x

Since the towns are 576 km apart, then;

3(x + 12) + 3x = 576

Divide through by 3 to get;

x + 12 + x = 192

2x + 12 = 192

2x = 192 - 12

2x = 180

x = 180/2

x = 90 km/h

Faster bus speed = 90 + 12 = 102 km/h

On a coordinate plane, rhombus W X Y Z has points (negative 3, 1), (1, 4), (5, 1), and (1, negative 2). Rhombus WXYZ is graphed on a coordinate plane. What is the area of the rhombus?

Answers

Answer:

B. 20 Units

Step-by-step explanation:

unit test review edge 2021

The area of rhombus WXYZ as shown in the diagram given with the given vertices is: 24 units².

What is the Area of a Rhombus?

Area of a rhombus = pq/2, where p and q are the diagonals of the rhombus.

Find the lengths of diagonals XZ and WY:

XZ = |4 -(-2)| = 6 units

WY = |-3 - 5| = 8 units

Area of rhombus WXYZ = 1/2(XZ × WY) = 1/2(6 × 8)

Area of rhombus WXYZ = 24 units².

Learn more about the area of a rhombus on:

https://brainly.com/question/2456096

#SPJ2

B
15x+7
6x+2y|
y +3
2y + 1
С
E
The triangles are congruent. Find the length of each hypotenuse.
A. 3
B. 5
C 17

Answers

Answer:

Hypothenus = 22

Step-by-step explanation:

From the question given above, we were told that the triangles are congruent (i.e same size). Thus,

AC = EF

BC = DE

To obtain the length of each Hypothenus, we shall determine the value of y and x. This can be obtained as follow:

For y:

AC = y + 3

EF = 2y + 1

AC = EF

y + 3 = 2y + 1

Collect like terms

3 – 1 = 2y – y

2 = y

y = 2

For x:

BC = 5x + 7

DE = 6x + 2y

y = 2

DE = 6x + 2(2)

DE = 6x + 4

BC = DE

5x + 7 = 6x + 4

Collect like terms

7 – 4 = 6x – 5x

3 = x

x = 3

Finally, we shall determine the length of each Hypothenus. This can be obtained as follow:

Hypothenus = BC

Hypothenus = 5x + 7

x = 3

Hypothenus = 5x + 7

Hypothenus = 5(3) + 7

Hypothenus = 15 + 7

Hypothenus = 22

OR

Hypothenus = DE

DE = 6x + 2y

y = 2

x = 3

Hypothenus = 6(3) + 2(2)

Hypothenus = 18 + 4

Hypothenus = 22

It cost $52 to use 800 kWh of electricity. How much will 650 kWh cost?

Answers

Hello!

$52 ..... 800kWh

$x ..... 650kWh

_____________

52/x = 800/650 <=>

<=> 52 × 650 = 800x <=>

<=> 33800 = 800x <=>

<=> 800x = 33800 <=>

<=> x = 33800/800 <=>

<=> x = 338/8 <=>

<=> x = 169/4 <=>

<=> x = 42,25$

Good luck! :)

A store offers packing and mailing services to customers. The cost of shipping a box is a combination of a flat packing fee of $5 and an amount based on the weight in pounds of the box, $2.25 per pound. Which equation represents the shipping cost as a function of x, the weight in pounds?

f(x) = 2.25x + 5
f(x) = 5x + 2.25
f(x) = 2.25x − 5
f(x) = 5x − 2.25

Answers

Answer:

f(x) = 2.25x + 5

Step-by-step explanation:

The cost from the weight in pounds of the box can be represented by 2.25x, since the charge is $2.25 per pound.

The flat fee of $5 can be represented in the function by adding 5 to 2.25x.

Put these together in function notation:

f(x) = 2.25x + 5

So, the equation is f(x) = 2.25x + 5

A psychologist suspects that heroin addicts have a different assessment of self-worth than others in the general population. On a test designed to measure self-worth, the mean for the general population is 48.6. The psychologist obtains a random sample of 40 test scores produced by heroin addicts and found the sample mean and the sample standard deviations are 45.5 and 8.5, respectively. Do these data indicate the self-worth of heroin addicts is less than that of the general population?

Answers

Answer:

The p-value of the test if 0.0131, which is less than the standard significance level of 0.05, meaning that these data indicates that the self-worth of heroin addicts is less than that of the general population.

Step-by-step explanation:

On a test designed to measure self-worth, the mean for the general population is 48.6.

At the null hypothesis, we test if the mean is of 48.6, that is:

[tex]H_0: \mu = 48.6[/tex]

A psychologist suspects that heroin addicts have a different assessment of self-worth than others in the general population. Test if it is lower.

At the alternative hypothesis, we test if the mean is lower, that is:

[tex]H_1: \mu < 48.6[/tex]

The test statistic is:

We have the standard deviation for the sample, which means that the t-distribution is used to solve this question.

[tex]t = \frac{X - \mu}{\frac{s}{\sqrt{n}}}[/tex]

In which X is the sample mean, [tex]\mu[/tex] is the value tested at the null hypothesis, s is the standard deviation and n is the size of the sample.

48.6 is tested at the null hypothesis:

This means that [tex]\mu = 48.6[/tex]

The psychologist obtains a random sample of 40 test scores produced by heroin addicts and found the sample mean and the sample standard deviations are 45.5 and 8.5, respectively.

This means that [tex]n = 40, X = 45.5, s = 8.5[/tex]

Value of the test statistic:

[tex]t = \frac{X - \mu}{\frac{s}{\sqrt{n}}}[/tex]

[tex]t = \frac{45.5 - 48.6}{\frac{8.5}{\sqrt{40}}}[/tex]

[tex]t = -2.31[/tex]

P-value of the test:

The p-value of the test is a one-tailed test(mean lower than a value), with t = -2.31 and 40 - 1 = 39 df.

Using a t-distribution calculator, this p-value is of 0.0131.

The p-value of the test if 0.0131, which is less than the standard significance level of 0.05, meaning that these data indicates that the self-worth of heroin addicts is less than that of the general population.

Enter the correct answer in the box.
Simplify the expression x^5•x^7

Answers

Answer:

[tex]x^{7}[/tex]

Step-by-step explanation:

When multiplying exponents, you add them together.

x^12

Step-by-step explanation:

x^5*x^7

x^5+7

=x^12

Simplify the given equation.
5x+ 2(x-3) = -2(x - 1)
0 7 x-6=-2 X-2
0 7 x - 6 = -2 x + 2
0 7 X - 3 = -2 x-1

i’m sorry:/

Answers

Firstly, we can distribute both sides, this gives us 5x + 2x - 6 = -2x + 2. Next, we can combine our like terms. 7x - 6 = -2x + 2. The second option, B, is our answer.

Answer:

7x - 6 = -2x + 2

Step-by-step explanation:

Hi! First we are going to distribute our "2" and "-2" values to the values in parenthesis.

5x + 2x - 6 = -2x + 2

Now, we can combine our like terms, "5x" and "2x".

7x - 6 = -2x + 2

Since our problem's answers are left in this form, not completely combined our answer is 7x - 6 = -2x + 2.

Hope this Helps! :)

Have any questions? Ask below in the comments and I will try my best to answer.

-SGO

Shirley has a collection of 50 stamps and adds 4 stamps daily to her collection. Model this situation as a function of number of days (d).

Answers

Answer:

N = 50 +4d

Step-by-step explanation:

Take the original number of stamps and add the stamps per days times the number of days

N = 50 +4d

I need help please. Show work

Answers

Answer:

28

Step-by-step explanation:

10/14 mph no wind

20 wind

14 x 2 = 28

28 mph with wind

Solve this application problem using a system of equations: A grocery store recently sold a
bag of peanuts for $0.76 and a bag of pistachios for $3.68. At the end of that day, 50 bags of
peanuts and pistachios were sold for a total of $128.52. How many bags of each were sold?

Answers

Answer:

19 bags of peanuts and 31 bags of pistachios were sold.

Step-by-step explanation:

This question is solved by a system of equations.

I am going to say that:

x is the number of bags of peanuts sold.

y is the number of bags of pistachios sold.

50 bags of  peanuts and pistachios were sold

This means that [tex]x + y = 50[/tex], that is: [tex]x = 50 - y[/tex]

A grocery store recently sold a  bag of peanuts for $0.76 and a bag of pistachios for $3.68. Were sold for a total of $128.52.

This means that:

[tex]0.76x + 3.68y = 128.52[/tex]

Since [tex]x = 50 - y[/tex]

[tex]0.76(50 - y) + 3.68y = 128.52[/tex]

[tex]2.92y = 90.5[/tex]

[tex]y = \frac{90.5}{2.92}[/tex]

[tex]y = 31[/tex]

[tex]x = 50 - y = 50 - 31 = 19[/tex]

19 bags of peanuts and 31 bags of pistachios were sold.

Uuannsnnsnndn d. DND. D

Answers

Answer:

im so confused

Step-by-step explanation:

Answer:

what is this goat saying

4. Write the number 3.8 in the form using integers, to show that it is a rational number. 8 11 38 10 10 38 100

Answers

Answer:

38 divided by 10 equals 3.8

find the differential equation of this function and indicate the order y = e^3x (acos3x +bsin3x)​

Answers

Answer:

y"-6y'+18y=0

Second order

Step-by-step explanation:

Since there are 2 constants, the order of the differential equation will be 2. This means we will need to differentiate twice.

y = e^(3x) (acos3x +bsin3x)​

y'=3e^(3x) (acos3x+bsin3x)

+e^(3x) (-3asin3x+3bcos3x)

Simplifying a bit by reordering and regrouping:

y'=e^(3x) cos3x (3a+3b)+e^(3x) sin3x (3b-3a)

y"=

3e^(3x) cos3x (3a+3b)+-3e^(3x) sin(3x) (3a+3b)

+3e^(3x) sin3x (3b-3a)+3e^(3x) cos(3x) (3b-3a)

Simplifying a bit by reordering and regrouping:

y"=

e^(3x) cos3x (9a+9b+9b-9a)

+e^(3x) sin3x (-9a-9b+9b-9a)

Combining like terms:

y"=

e^(3x) cos3x (18b)

+e^(3x) sin3x (-18a)

Let's reorder y like we did y' and y".

y = e^(3x) (acos3x +bsin3x)

y=e^(3x) cos3x (a) + e^(3x) sin3x (b)

Objective is to find a way to combine or combine constant multiples of y, y', and y" so that a and b are not appearing.

Let's start with the highest order derivative and work down

y"=

e^(3x) cos3x (18b)

+e^(3x) sin3x (-18a)

We need to get rid of the 18b and 18a.

This is what we had for y':

y'=e^(3x) cos3x (3a+3b)+e^(3x) sin3x (3b-3a)

Multiplying this by -6 would get rid of the 18b and 18a in y" if we add them.

So we have y"-6y'=

e^(3x) cos3x (-18a)+e^(3x) sin3x (-18b)

Now multiplying

y=e^(3x) cos3x (a) + e^(3x) sin3x (b)

by 18 and then adding that result to the y"-6y' will eliminate the -18a and -18b

y"-6y'+18y=0

Also the characteristic equation is:

r^2-6r+18=0

This can be solved with completing square or quadratic formula.

I will do completing the square:

r^2-6r+18=0

Subtract 9 on both sides:

r^2-6r+9=-9

Factor left side:

(r-3)^2=-9

Take square root of both sides:

r-3=-3i or r-3=3i

Add 3 on both sides for each:

r=3-3i or r=3+3i

This confirms our solution.

Another way to think about the problem:

Any differential equation whose solution winds up in the form y=e^(px) (acos(qx)+bsin(qx)) will be second order and you can go to trying to figure out the quadratic to solve that leads to solution r=p +/- qi

Note: +/- means plus or minus

So we would be looking for a quadratic equation whose solution was r=3 ×/- 3i

Subtracting 3 on both sides gives:

r-3= +/- 3i

Squaring both sides gives:

(r-3)^2=-9

Applying the exponent on the binomial gives:

r^2-6r+9=-9

Adding 9 on both sides gives:

r^2-6r+18=0

What's the equivalent expression.
(2-7. 5)² =?

Answers

Answer:

The Answer of the above question is 30.25

Step-by-step explanation:

Hope it helps you.

Answer: 30.25


Step 1: Rewrite equation

To start off, I find it helpful to rewrite the equation. This will give you a better understanding of the problem. Let’s do this now.

(2-7.5)²

Step 2: Understand PEMDAS

Now that we have written down the equation, let’s solve using PEMDAS. In case you are unfamiliar, I will write down what each letter stands for. Keep in mind that the letters go in the order you need to solve.

P- Parentheses
E- Exponent
M- Multiplication
D- Division
A- Addition
S- Subtraction

Step 3: Solve

Finally, we can solve the equation. Using the knowledge from step 2, we will be able to solve in the correct order. Let’s start with parentheses, and find the answer for 2-7.5.

(2-7.5)²
-5.5²

Now let’s move on to the next letter and solve using the exponent. We will multiply -5.5 by itself. This will be the final step; although there are other letters in PEMDAS, the equation will end after this step since we will already have the answer.

-5.5²
30.25


Hope this helps! Comment below for more questions.

please help, it’s urgent !!!

Answers

Answer:

1. purple graph

2. orange graph

3. green graph

4. blue graph

mmmmmmmm ur mom!!’my moma a mild

What is the percent discount if a 12,500 car is now on special for 10,250?

Answers

Answer:

Step-by-step explanation:

the answer is 2250 percent is = 22.5

Answer:

18% discount

Step-by-step explanation:

Percent discount is found by the following formula:

[tex]\frac{original-discount}{original}[/tex]

In this scenario, the original is 12500 and the discount, or special is 10250.

We can plug this into the formula to get

[tex]\frac{12500-10250}{12500}[/tex]

We can simplify the numerator by subtracting, and we get that answer as 2250.

We get the remainder of the answer as 2250 divided by 12500. We divide that, and get the answer as 0.18, which can be rewritten as 18%.

Match each equation to its graph.

1. y= x-2
2. y= -2x
3. x= -2
4. y= -2

Answers

1 equation goes with the graph one, 2nd equation goes with 4th graph , 3rd equation goes with 3rd graph, and 4th equation goes with graph graph 2.

If the equation is...

y=c, where c is a constant, the line is horizontal across that c value, on y axis. x=c the line would be vertical across that c, on x axis.

The equation of the graph is : x = -2

What is a graph?

A graph is a pictorial representation of the locus of a certain point.

How to draw a graph?

A graph can be drawn by picking some fixed points from the locus of the point.

Here, the straight line passes through the points are : (-2,1); (-2,2); (-2,-5);(-2,-5).

Hence, the straight line is to be parallel to the x-axis and the equation of the graph is x= -2.

Learn more about graph here :

https://brainly.ph/question/6429575

#SPJ2

Rewrite the following expanded notation in standard form. 600,000 + 80,000 + 1,000 + 400 + 70 + 5

Answers

Answer:

this is the answer

681,474

the answer is 681,475
Other Questions
What type of person are called the persons of highest grade? Instructions: Find the missing segment in the image below.Triangle with a missing segment. Diane bought new headphones originally listed for $70.99. They are 25% off. Which equation can be used to find the amount Diane will save? The Wisconsin Lottery will pay a lottery winner a lump sum payment of $19,046,180 as the final payment of her winnings in four years. If the appropriate discount rate for the payment is 8.6% what is the present value of the payment?a. $5,191,977.b. $5,408,309.c. $116,741.d. $17,899,197.e. $17,899,197. Now suppose that not every player can play in every position. The outfielders (left field, center field, right field) can play any outfield position, the infielders (1st base, 2nd base, 3rd base, short stop) can play any infield position, the pitchers can only pitch, and the catchers can only catch. Suppose a certain team has 20 players, of whom 3 are catchers, 4 are outfielders, 6 are infielders, and 7 are pitchers.How many ways can the team assign field positions to 9 of the 19 players, putting each of the 9 selected players in a position he can play, and ensuring that all 9 field positions are filled? As you read the paragraph below, think about how you would describe the main character's reaction to the event.Paula had not seen her grandfather for over a year. He had been too sick to travel from his home in New York to see Paula's family in California, though her mother had been to New York a few times. Now Paula just wanted to give her grandfather a huge hug and not let go. And finally he was here, walking up her driveway.Which next sentence provides the most precise description of Paula's reaction?A Paula was pleased to see her grandfather again.B Paula was excited to see her grandfather again.C Paula was content to see her grandfather again.D Paula was thrilled to see her grandfather again. 12. What would be the other strand of DNA?-A T C C G A A GTC C GA G- How the aging is related with the methylation? The methylation increase or decreased ? If the electric force between two charges is 4.2 10-2 N, what would the new force be if the distance between the charges is doubled and the charge on one of the charges is tripled? What is the purpose of a standard drink measurement? It allows you to determine if you're getting what you paid for It is a consistent way of measuring the amount of alcohol in a drink It allows you to determine what is inside of a mixed drink It is a measurement to determine if you are legally drunk Which pair of angles are corresponding angles? Suppose you had d dollars in your bank account. You spent $22 but have at least $28 left. How much money did you have initially? Write and solve an inequality thatrepresents this situation.a.) d-22 28; d 50b.) d - 22 > 28; d > 50c.) d + 22 s 28; d s 72d.) d + 22 28; d 272 Joe has an idea for a new mobile restaurant business. He wants to convert an antique bus into a sit-down restaurant with a service window allowing him to serve people within the bus and walk-ups who want to get their food and take it home. Joe takes his idea and looks at the people, the offerings, the customers, the value proposition, and his distinctive competencies. Joe is performing a(n) ______A. business model canvas B. RBI screen C. classic feasibility study D. pilot test Why did Rutherford choose alpha particles in his experiment? help me ):Sexual DisordersName and describe a sexual disorder that occurs in Men. Name and describe a sexual disorder that occurs in Women.Identify and describe three causes of sexual disorders. how to work this problem 736.5 x .78 The mass of the sun is 2.1352510^30 kilograms. The mass of Mercury is 3.28510^23 kilograms.How many times greater is the mass of the sun than the mass of Mercury?Type your answer in scientific notation form. With malice toward none with charity for all what is the conclusion of Lincoln speech If angle ACB is congruent, and angle BAC=3x-10, angle ECD=45 and angle EDC=2x+10 what is x? Which key details should be included in a paraphrase of this passage? Check all that apply.The storm was a direct result of Zeuss fury at the men.For two days and nights, Zeus created a storm at sea.Odysseus and his men feared greatly for their lives.The sails on the ship cracked in the heavy winds.The men felt grief as they lay offshore and waited.